APEA Ortho Assignment

APEA Ortho Assignment

Question:

A tool for assessing risk factors for osteoporotic fractures is the:

 

DEXA.FRAX.  CorrectBRCA1.HAARM.  Incorrect

 

Explanation:

The FRAX calculator generates fracture risk based on age, body mass index, parental fracture history, use of glucocorticoids, presence of rheumatoid arthritis or secondary osteoporosis, and tobacco and alcohol use. It has been validated for black, Hispanic, and Asian women in the USA and has calculators that are country and continent specific. Duel energy x-ray absorptiometry, DEXA, is the optimal standard for measuring bone density. APEA Ortho. BRAC1 is a gene that can mutate and increase the risk of breast cancer. HAARM is the melanoma risk model. APEA Ortho Assignment

Question:

Anserine bursitis arises from:

 

excessive running.  Correctexcessive kneeling.  Incorrectarthritis.trauma

 

Explanation:

Anserine bursitis arises from excessive running, valgus knee deformity, fibromyalgias, and osteoarthritis. Prepatellar bursitis (“housemaid’s knee”) arises from excessive kneeling. A popliteal or “baker’s” cyst arises from distention of the gastrocnemius semimembranous bursa from underlying arthritis or trauma. APEA Ortho.

Question:

When examining the knee, which of the following symptoms could be indicative of a positive Adduction (Varus) Stress Test?

 

Pain in the lateral joint line  CorrectPain in the medial joint line  IncorrectPain in the anterior joint lineA click along the medial joint line.

 

Explanation:

The Adduction (or Varus) Stress Test is a maneuver that evaluates the function of the lateral collateral ligament. To perform this test, the knee is held in 30 degrees of flexion. With one hand on the medial side of the knee and one hand on the ankle, an adduction force is gently applied. If pain is noted in the lateral joint line, this could be indicative of a lateral collateral ligament tear. When tenderness extends more to the proximal or distal joint line, the collateral ligament may be the cause of pain instead of the meniscus  APEA Ortho Assignment.

Question:

When assessing the knee, the examiner instructs the patient to straighten his knee. This motion would assess knee:

 

flexion.extension.  Correctinternal rotation.external rotation.

 

Explanation:

Having the patient straighten his leg assesses extension of the knee. The examiner instructs the patient to bend his knee. APEA Ortho. This maneuver assesses knee flexion. Internal rotation of the knee could be elicited by having the patient swing his lower leg toward the midline while sitting. Instructing the patient to swing his leg away from his midline while sitting assesses external rotation of the knee.

ORDER   A PLAGIARISM FREE PAPER   NOW

Question:

When performing an examination of a tender left finger on an adult, the surrounding tissue reveals warmth, edema, and redness. This finding could be suggestive of:

 

carcinoma.muscular atrophy. synovitis.  Incorrectgouty arthritis.  Correct

 

Explanation:

Redness, warmth, and edema over a tender joint suggest septic or gouty arthritis infection, or possibly rheumatoid arthritis.

Question:

An example of a fibrous joint would be the:

 

vertebral bodies of the spine.skull.   Correctshoulder.  Incorrectpubic symphysis of the pelvis.

 

Explanation:

The skull is an example of the fibrous joint. Examples of synovial joints include the shoulder, knee, hip, wrist, distal radioulnar, elbow, and carpals. Vertebral bodies of the spine and the pubic symphysis of the pelvis are examples of cartilaginous joints  APEA Ortho Assignment

Question:

To palpate the medial meniscus, slightly internally rotate the tibia and palpate the medial soft tissue along the:

 

lateral joint line of the knee.on either side of the patella.upper edge of the tibial plateau.  Correcttop of the patella.

 

Explanation:

To palpate the medial meniscus, slightly internally rotate the tibia and palpate the medial soft tissue along the upper edge of the tibial plateau. The lateral meniscus is palpated on the lateral joint line by placing the patient’s knee in slight flexion. To palpate the tibiofemoral joint, face the patient’s knee and place the thumbs in the soft-tissue depressions on either side of the patellar tendon.

Question:

A patient complains of a sharp burning pain in the neck and right arm with associated paresthesias and weakness. These symptoms may be associated with:

 

mechanical neck pain.mechanical neck pain with whiplash.cervical radiculopathy.  Correctcervical myelopathy.

 

Explanation:

With cervical radiculopathy, nerve root compression is the etiology. Symptoms may include sharp burning or tingling pain in the neck and one arm with associated paresthesias. Mechanical neck pain is described as aching pain in the cervical paraspinal muscles and ligaments with associated muscle spasm and stiffness and tightness in the upper back and shoulder, lasting up to 6 weeks. In patients with mechanical neck pain with whiplash, the paracervical pain and stiffness begins the day after injury and may be accompanied by occipital headaches, dizziness, and malaise. In cervical myelopathy, cervical cord compression, the neck pain is associated with bilateral weakness and paresthesias in both upper and lower extremities.

Question:

A patient complains of lateral hip pain while pointing near the trochanter. This type of pain could be suggestive of:

 

sciatica.  Incorrectradicular pain.polyarticular arthritis.bursitis.  Correct

 

Explanation:

Lateral hip pain near the greater trochanter suggests trochanteric bursitis. Sciatica symptoms usually include a shooting pain below the knee, commonly in the lateral leg or posterior calf and accompanied by low back pain. Radicular pain refers to pain that radiates along the dermatome of a nerve due to inflammation or irritation of a nerve root, as with sciatica pain. Polyarticular arthritis refers to arthritis involving several joints.

Question:

The muscle of the scapulohumeral group that crosses the glenohumeral joint posteriorly and inserts on the greater tubercle is known as the:

 

infraspinatus muscle.  Correctpectoralis major. subscapularis muscle.  Incorrectsupraspinatus muscle.

 

Explanation:

One of the muscles of the scapulohumeral group that crosses the glenohumeral joint posteriorly and inserts on the greater tubercle is the infraspinatus muscle. The other one is the teres minor muscle. The pectoralis major muscle is situated on the anterior chest. The muscle that runs above the glenohumeral joint and inserts on the greater tubercle is known as the supraspinatus. The subscapularis muscle originates on the anterior surface of the scapula and crosses the joint anteriorly and inserts on the lesser tubercle.

Question:

Inspection of the hip begins with careful observation of a patient’s gait. A patient’s foot moves forward without bearing weight. This is known as the: APEA Ortho Assignment.

 

swing phase of gait.  Correctstance phase of gait.push off phase of gait.heel strike phase of gait.

 

Explanation:

Inspection of the hip begins with careful observation of a patient’s gait. There are 2 phases of gait: stance and swing. The swing phase occurs when the foot moves forward and does not bear weight. The stance phase occurs when the foot is on the ground bearing weight.

Question:

The axioscapular group of muscles:

 

pulls the shoulder backward.  Correctrotates the shoulder laterally.produce internal rotation of the shoulder.  Incorrectdraws the shoulder blade forward.

 

Explanation:

The axioscapular group pulls the shoulder backward and rotates the scapula. The scapulohumeral group of muscles rotates the shoulder laterally, including the rotator cuff, and depresses and rotates the head of the humerus. The axiohumeral group produces internal rotation of the shoulder. The serratus anterior draws the shoulder blade forward.

Question:

When assessing the knee, the examiner instructs the patient to sit and swing his lower leg toward midline. This motion assesses knee:

 

flexion.extension.internal rotation.  Correctexternal rotation.

 

Explanation:

Internal rotation of the knee is elicited by having the patient swing his lower leg toward the midline while sitting. Instructing the patient to bend his knee assesses knee flexion. Having the patient straighten his leg assesses extension of the knee. APEA Ortho. Instructing the patient to swing his leg away from his midline while sitting would be a maneuver to assess external rotation of the knee.

Question:

When performing a musculoskeletal examination, the nurse practitioner instructs the patient to move his arm in front of his body. This motion of the shoulder girdle would be an example of:

 

adduction.  Incorrectabduction.flexion.  Correctextension.

 

Explanation:

When performing a musculoskeletal examination, the nurse practitioner instructs the patient to move his arm in front of his body. This motion of the shoulder girdle would be an example of flexion. Extension occurs when the patient moves his arm behind himself. Abduction occurs when the patient moves his arm away from the body laterally and overhead. Adduction occurs when the patient moves his arm across his body.

Question:

Static stabilizers of the shoulder are referred to as those structures that are:

 

muscular structures of the shoulder girdle.capable of movement. bony structures of the shoulder girdle.  Correctresponsible for stabilizing the humeral head in the glenoid cavity.  Incorrect

 

Explanation:

Static stabilizers are incapable of movement and include the bony structures of the shoulder girdle, the labrum, the articular capsule, and the glenohumeral ligaments that add to joint stability. Dynamic stabilizers are capable of movement and include the SITS muscles of the rotator cuff (supraspinatus, infraspinatus, teres minor, and subscapularis). These muscles move the humerus and compress and stabilize the humeral head within the glenoid cavity.

Question:

To test the thumb for abduction, ask the patient to:

 

move his thumb across his palm and touch the base of the fifth finger.  Incorrectmove his thumb from the base of the fifth finger and then as far away from the palm as possible.touch the thumb to each of the other fingertips. place the fingers and thumbs in the neutral position with the palm up and then move the thumb anteriorly away from the palm.  Correct

 

Explanation:

Placing the fingers and thumbs in the neutral position with the palm up and moving the thumb anteriorly away from the palm assesses abduction. To test the thumb for flexion, ask the patient to move his thumb to touch the base of the fifth finger. To test extension, ask the patient to move his thumb from the base of the fifth finger, across the palm, and then as far away from the palm as possible. APEA Ortho. Touching the thumb to each of the other fingers tests opposition. Moving the thumb back to its neutral position assesses adduction.

Question:

Passive flexion, valgus stress, and internal rotation of the lower leg, evaluates the:

 

medial meniscus.lateral meniscus.  Correctlateral collateral ligament (LCL).  Incorrectposterior cruciate ligament (PCL).

 

Explanation:

Passive flexion, valgus stress, and internal rotation of the lower leg, evaluates the lateral meniscus.

Question:

The nurse practitioner instructs the patient to lie supine, bend his knee, and turn his lower leg and foot away from the midline. This maneuver would assess hip:

 

abductionextension.external rotation.  Incorrectinternal rotation.  Correct

 

Explanation:

To assess for hip internal rotation, the patient would lie supine, bend his knee, and turn his lower leg and foot away from the midline. To assess hip abduction, the patient would lie supine and move his lower leg away from the midline. To assess hip extension, the patient would lie face up, bend his knees, place feet flat on the table, and lift his buttocks off the table. To assess for external rotation of the hip, the patient would lie supine, bend his knee, and turn the lower leg and foot toward the midline.  APEA Ortho Assignment

Question:

Children with Legg-Calve Perthes disease should:

 

maintain a diet high in protein, vitamins and minerals.sleep on a firm mattress to prevent contractures.   Incorrectavoid weight bearing on the affected extremity.  Correctbe allowed to play basketball.

 

Explanation:

Legg–Calvé–Perthes disease is a childhood hip disorder initiated by a disruption of blood flow to the head of the femur. Due to the lack of blood flow, the bone dies (osteonecrosis or avascular necrosis) and stops growing. Over time, healing occurs by new blood vessels infiltrating the dead bone and removing the necrotic bone which leads to a loss of bone mass and a weakening of the femoral head. APEA Ortho. The bone loss leads to some degree of collapse and deformity of the femoral head and sometimes secondary changes to the shape of the hip socket. The goals of treatment are to decrease pain, reduce the loss of hip motion, and prevent or minimize permanent femoral head deformity so that the risk of developing a severe degenerative arthritis as an adult can be reduced. Diet and sleeping on a firm mattress do not alter the course of the disease. However, avoiding high impact sports such as basketball during treatment is essential since increased weight on the hip will cause further damage.

Question:

The convex medial end of the clavicle that articulates with the concave hollow in the upper sternum is referred to as the:

 

glenohumeral joint.sternoclavicular joint.  Correctacromioclavicular joint.manubrium joint.  Incorrect

 

Explanation:

The convex medial end of the clavicle articulates with the concave hollow in the upper sternum to form the sternoclavicular joint. The glenohumeral joint is where the head of the humerus articulates with the shallow glenoid fossa of the scapula. This joint is deeply situated and not normally palpable. The acromioclavicular joint lies at the lateral end of the clavicle and articulates with the acromion process of the scapula. There is no manubrium joint; it is the broad upper part of the sternum.

Question:

When assessing a patient with complaints consistent with carpal tunnel syndrome, which one of the following symptoms is unlikely?

 

Dropping objectsInability to twist lids off jarsTingling of the first three digits of the handNumbness of the last two digits of the hand  Correct

 

Explanation:

For complaints of dropping objects, inability to twist lids off jars, aching at the wrist or even the forearm, and numbness of the first three digits, consider carpal tunnel syndrome.

Question:

When evaluating a patient who complains of thumb pain, the nurse practitioner would test thumb movement by instructing the patient to place his thumb in the palm and then move the wrist toward the midline in ulnar deviation. This maneuver is commonly known as:

 

de Quervain’s test.Finkelstein’s test.  CorrectTinel’s test.Phalen’s test.

 

Explanation:

When evaluating a patient who complains of thumb pain, the nurse practitioner would test thumb movement by instructing the patient to place his thumb in his palm and then move the wrist toward the midline in ulnar deviation. This maneuver is commonly known as Finkelstein’s test. Tinel’s test assesses for median nerve compression. The examiner would tap lightly over the median nerve of the carpal tunnel. To test Phalen’s sign, the patient would hold his wrists in flexion for 60 seconds while pressing the backs of his hands together to form right angles APEA Ortho.

Question:

Olecranon bursitis may be caused by all of the following except:

 

gout.trauma.frozen shoulder.  Correctosteoarthritis.

 

Explanation:

Olecranon bursitis refers to swelling and inflammation of the olecranon bursa and may result from trauma, gout, or arthritis.

Question:

Women who wear high-heeled shoes with narrow toe boxes are at risk of developing all of the following forefoot abnormalities except:

 

hallux valgus.metatarsalgia.Achilles tendinitis.  CorrectMorton’s neuroma.  Incorrect

 

Explanation:

Women who wear high-heeled shoes with narrow toe boxes are at risk of developing hallux valgus, metatarsalgia, and Morton’s neuroma. Achilles tendinitis more commonly occurs in runners and affects the posterior foot as opposed to the forefoot.

Question:

Upon examination of the foot and ankle, the nurse practitioner notes point tenderness over the posterior aspects of the right malleolus. Additionally, the patient is unable to bear weight after 4 steps. This finding is most consistent with:

 

Achilles tendinitis.  Incorrectan ankle fracture.   Correcta ligamentous injury.rheumatoid arthritis.

 

Explanation:

Point tenderness over the posterior aspects of the right malleolus with an inability to bear weight after 4 steps could be consistent with an ankle fracture. Rheumatoid nodules and tenderness may be associated with Achilles tendinitis. Localized tenderness on examination of the ankle joint could be suggestive of arthritis, infection of the ankle, or ligamentous injury. Tenderness on compression of the foot is an early sign of rheumatoid arthritis.

Question:

The small intrinsic muscles are located:

 

at the anterior surface of the vertebrae. on either side of the midline of the vertebrae.  Incorrectbetween the vertebrae.  Correctin front of the cervical vertebrae.

 

Explanation:

The small intrinsic muscles are located between the vertebrae. Prevertebral muscles run in front of the cervical vertebrae, and they contract generally to flex the neck and bow the head. The paravertebral muscles are located on both sides of the vertebrae and extend downward to the entire spine. Muscles that attach to the anterior surface of the vertebrae include the psoas muscles and the muscles of the abdominal wall. APEA Ortho.

Question:

The nurse practitioner instructs the patient look upward at the ceiling. This maneuver assesses cervical:

 

flexion.extension.  Correctrotation.lateral bending.

 

Explanation:

Assessing neck extension occurs by asking the patient to look upward at the ceiling. Bringing the chin to the chest assesses flexion of the cervical spine. Looking over one shoulder and then the other assesses rotation of the neck. Asking the patient to bring his ear to his shoulder assesses lateral bending of the cervical spine.

Question:

Upon examination of the left shoulder, the patient complains of a dull, aching pain when attempting active or passive range of motion and localized tenderness with external rotation. These symptoms could be suggestive of:

 

a complete rotator cuff tear.adhesive capsulitis.  Correctrotator cuff tendinitis.  Incorrectcalcific tendinitis.

 

Explanation:

Adhesive capsulitis, or frozen shoulder, refers to fibrosis of the glenohumeral joint capsule resulting in a dull aching pain in the shoulder. It progresses to restriction of active and passive range of motion and tenderness with external rotation. With complete rotator cuff tears, active abduction and forward flexion at the glenohumeral joint are severely impaired. A characteristic shrug of the shoulder is noted with a positive arm drop on the affected side. Reports of sharp “catches” of pain, grating, and weakness in the shoulder when lifting the arm overhead are symptoms suggestive of rotator cuff tendinitis or impingement syndrome. Calcific tendinitis involves the supraspinatus tendon and is associated with deposition of calcium salts. This results in disabling attacks of shoulder pain severely limiting motions due to the pain.

Question:

The area at the posterior aspect of the spine lateral to the sacroiliac joint is known as the:

 

posterior superior iliac spine.  Correctischial tuberosity.superior ramus of pubis.pubic symphysis.

 

Explanation:

The posterior superior aspect of the spine lateral to the sacroiliac joint is known as the posterior superior iliac spine. The ischial tuberosity is a large swelling posteriorly on the superior ramus of the ischium. It marks the lateral boundary of the pelvic outlet and bares most of the weight when sitting. The superior ramus of pubis are the pubic bones that help form the obturator foramen. The pubic symphysis is a cartilage-like articulation between the pubic bones.

Question:

Which nerve in the arm runs posteriorly in the ulnar groove between the medial epicondyle and the olecranon process?

 

Median nerveUlnar nerve  CorrectRadial nerveBrachial plexus

 

Explanation:

The ulnar nerve runs posteriorly in the ulnar groove between the medial epicondyle and the olecranon process. The median nerve is located on the ventral forearm and is just medial to the brachial artery in the antecubital fossa. APEA Ortho. The radial nerve originates in the axilla and travels down the arm in a shallow depression (radial groove) on the surface of the humerus. The brachial plexus runs from the spine through the neck, the axilla, and into the arm.

Question:

When examining the knee, the presence of a palpable fluid wave with the returning fluid wave into the suprapatellar pouch is noted. This positive sign for effusion of the knee is known as the:

 

balloon sign.  Correctbulge sign.  Incorrectballoting sign.McMurray’s sign.

 

Explanation:

A positive balloon sign for effusion in the knee is the presence of a palpable fluid wave with a returning fluid wave into suprapatellar pouch. When examining the knee, a fluid wave on the medial side between the patella and the femur is noted. This positive sign for effusion is known as the bulge sign. Balloting of the patella is tested by compressing the suprapatellar pouch and pushing the patella sharply against the femur. If fluid returns to the suprapatellar pouch, then an effusion of the knee is diagnosed. McMurray’s test checks for tears in the medial meniscus.

Question:

Dynamic stabilizers of the shoulder are referred to as those structures that are:

 

incapable of movement.capable of movement.   Correctbony structures of the shoulder girdle.  Incorrectresponsible for joint stability.

 

Explanation:

Dynamic stabilizers are capable of movement and include the SITS muscles of the rotator cuff (supraspinatus, infraspinatus, teres minor, and subscapularis). These muscles move the humerus and compress and stabilize the humeral head within the glenoid cavity. Static stabilizers are incapable of movement and include the bony structures of the shoulder girdle, the labrum, the articular capsule, and the glenohumeral ligaments that add to joint stability.

Question:

A structural channel beneath the palmar surface of the wrist and proximal hand is known as the:

 

median nerve plexus.carpal tunnel.  Correctcarpal sheath.flexor retinaculum.

 

Explanation:

A structural channel beneath the palmar surface of the wrist and proximal hand is known as the carpal tunnel. The carpal sheath covers the tendons. The flexor retinaculum is the transverse ligament that holds the sheath in place. APEA Ortho Assignment

Question:

When describing muscle strength, the term paraplegia means:

 

impaired strength.absence of strength.paralysis of all four extremities.paralysis of the legs.  Correct

 

Explanation:

Paraplegia means paralysis of the legs. Impaired strength is called weakness, or paresis. Absence of strength is called paralysis, or plegia. Quadriplegia means inability to move or paralysis of all four limbs.

Question:

A patient reports sharp “catches” of pain, grating, and weakness in the right shoulder when lifting the arm overhead. These symptoms could be suggestive of:

 

a complete rotator cuff tear.adhesive capsulitis.rotator cuff tendinitis.  Correctcalcific tendinitis.

 

Explanation:

Reports of sharp “catches” of pain, grating, and weakness in the shoulder when lifting the arm overhead are symptoms suggestive of rotator cuff tendinitis or impingement syndrome. With complete rotator cuff tears, active abduction and forward flexion at the glenohumeral joint are severely impaired. A characteristic shrug of the shoulder is noted with a positive arm drop on the affected side. Adhesive capsulitis, or frozen shoulder, refers to fibrosis of the glenohumeral joint capsule resulting in a dull aching pain in the shoulder. APEA Ortho. It progresses to restriction of active and passive range of motion. Calcific tendinitis involves the supraspinatus tendon and is associated with deposition of calcium salts. This results in disabling attacks of shoulder pain severely limiting motions due to the pain.

Question:

The structure that encloses the spinal cord is known as the:

 

articular process.spinous process.articular facets.vertebral foramen.  Correct

 

Explanation:

The vertebral foramen encloses the spinal cord. The structure that projects from the spinal column posteriorly in the midline is referred to as the spinous process. The articular processes are located on each side of the vertebra at the junction of the pedicles and the laminae, also referred to as the articular facets.

Question:

When examining the patient for wrist adduction, the nurse practitioner instructs the patient with his palms down to:

 

point his fingers toward the ceiling. move his fingers toward the midline.   Correctmove his fingers away from the midline.point his fingers toward the floor.   Incorrect

ORDER   A PLAGIARISM FREE PAPER   NOW

Explanation:

Adduction occurs by moving fingers toward the midline. When examining the patient for wrist flexion, the nurse practitioner instructs the patient to position his palms down and to point his fingers toward the floor. Extension occurs with pointing fingers toward the ceiling. Abduction occurs by having the patient bring his fingers away from the midline.

Question:

An example of a cartilaginous joint would be the:

 

vertebral bodies of the spine.  Correctskull. shoulder.knee.

 

Explanation:

Vertebral bodies of the spine and the pubic symphysis of the pelvis are examples of cartilaginous joints. Examples of synovial joints include the shoulder, knee, hip, wrist, distal radioulnar, elbow, and carpals. The skull is an example of the fibrous joint.

Question:

When grading muscle strength, a five would indicate:

 

no muscular contraction detected.barely detectable trace of contraction.active movement of the body part with gravity eliminated.active movement against full resistance without fatigue APEA Ortho.  Correct

 

Explanation:

A grade of five for muscle strength would indicate active movement against full resistance without fatigue. Zero muscular strength would indicate no muscular contraction was noted on exam. A grade of one indicates a barely detectable trace of contraction noted on exam. For active movement of the body part with gravity eliminated, a grade of two would be noted.

Question:

With the patient in the dorsal decubitus position, have him slowly extend the knee while maintaining the varus stress and external rotation. If a snap on the medial joint line is palpated, this may indicate a positive test for a:

 

lateral collateral ligament (LCL) tear.medial collateral ligament (MCL) tear.posterior cruciate ligament (PCL) tear.medial meniscal tear.  Correct

 

Explanation:

With the patient in the dorsal decubitus position, have him slowly extend the knee while maintaining the varus stress and external rotation. If a snap on the medial joint line is palpated, this may indicate a medial meniscal tear or a positive McMurray test. The Adduction (Varus) Stress Test evaluates the function of the lateral collateral ligament, while the Abduction (Valgus) Stress Test evaluates the medial collateral ligament APEA Ortho. To test the posterior cruciate ligament, the posterior drawer sign would be assessed.

Question:

When examining the elbow for range of motion, the nurse practitioner instructs the patient to bend his elbow. This motion is an example of:

 

extension.flexion.  Correctsupination.pronation.

 

Explanation:

Instructing the patient to bend his elbow is an example of flexion. Extension occurs with straightening the elbow. Turning the palms upward demonstrate supination. Turning the palms downward demonstrates pronation.

Question:

The nurse practitioner instructs the patient to move his ear to his shoulder. This maneuver assesses:

 

cervical flexion.cervical extension.rotation.lateral bending.  Correct

 

Explanation:

Looking over one shoulder and then the other would be assessing rotation of the neck and having the patient bring his ear to his shoulder would be assessing lateral bending of the neck. Assessing neck extension would be having the patient look upward at the ceiling. Bringing the chin to the chest would be assessing flexion of the neck.

Question:

The nurse practitioner instructs the patient to move his extended fingers so that each touches its nearest finger. This motion assesses the fingers and thumbs for:

 

adduction.  Correctabduction.flexion.  Incorrectextension.

 

Explanation:

Instructing the patient to make a fist with the left hand and to place his left thumb on top of the distal fingers assesses flexion. Extension occurs when the patient is able to completely move the fingers away from the palm and fingers are most distal from the palm. Asking the patient to move the fingers as far apart from each other as possible demonstrates abduction. The fingers must be kept in the same plane. The ability to move the fingers so that each digit touches the finger next to it assesses adduction. The fingers must be kept in the same plane.

Question:

The nurse practitioner would tap lightly over the median nerve in the carpal tunnel to assess:

 

Finkelstein’s test.Tinel’s test.  CorrectPhalen’s test.  Incorrectthumb abduction.

 

Explanation:

The examiner taps lightly over the median nerve in the carpal tunnel to assess Tinel’s sign. A positive Tinel’s test may indicate carpal tunnel syndrome. Asking the patient to point his thumb upward while the examiner applies downward resistance tests thumb abduction. To test thumb movement, instruct the patient to grasp his thumb against his palm and then move his wrist toward the midline in ulnar deviation. This maneuver is commonly known as Finkelstein’s test. To test Phalen’s sign, the patient would hold his wrists in flexion for 60 seconds while pressing the backs of his hands together to form right angles.

Question:

Which one of the following conditions can plantar fasciitis be associated?

 

Achilles tendinitisAn ankle fracture A ligamentous injuryRheumatoid arthritis  Correct

 

Explanation:

Focal heel tenderness on palpation of the plantar fascia suggests plantar fasciitis. This condition can be seen in prolonged standing or heel-strike exercise and also in rheumatoid arthritis, and gout.

Question:

A patient complains of shooting pains below the knee radiating into the lateral leg and calf. This type of low back pain is referred to as:

 

radicular low back pain.  Correctmechanical low back pain.lumbar spinal stenosis.pseudoclaudication pain.  Incorrect

 

Explanation:

Radicular low back pain, or sciatica, presents with shooting pains below the knee, into the lateral leg or posterior calf. It may be accompanied by paresthesias and/or weakness in the affected leg. Mechanical low back pain often arises from muscle and ligament injuries (~70%) or age-related intervertebral disc or facet disease. Common symptoms include aching pain in the lumbosacral area that radiates to the upper leg. APEA Ortho. Common risk factors include heavy lifting, poor conditioning, and obesity. Lumbar spinal stenosis or “pseudoclaudication” refers to pain in the back or legs with walking that improves with rest, lumbar flexion, or both.

Question:

Where the head of the humerus articulates with the shallow glenoid fossa of the scapula is known as the:

 

glenohumeral joint.  Correctsternoclavicular joint.acromioclavicular joint.  Incorrectmanubrium joint.

 

Explanation:

The glenohumeral joint is where the head of the humerus articulates with the shallow glenoid fossa of the scapula. This joint is deeply situated and not normally palpable. The acromioclavicular joint lies at the lateral end of the clavicle and articulates with the acromion process of the scapula. The convex medial end of the clavicle articulates with the concave hollow in the upper sternum to form the sternoclavicular joint. There is no manubrium joint; it is the broad upper part of the sternum.

Question:

Which of the following symptoms would be suggestive of lumbar spinal stenosis?

 

Calf wastingThigh pain after 30 seconds of lumbar extension  CorrectAbsent ankle jerkLoss of normal lumbar lordosis

 

Explanation:

Lumbar spinal stenosis arises from hypertrophic degenerative disease of one or more vertebral facets and thickening of the ligamentum flavum, causing narrowing of the spinal canal. Symptoms include posture flexed forward, lower extremity weakness, hyporeflexia, and thigh pain after 30 seconds of lumbar extension. Calf wasting and absent ankle jerk are typically noted in patients with sciatica or disc disease. Loss of the normal lumbar lordosis is a common cause of low back pain.

Question:

When inspecting the face, asymmetry is noted. This finding could be suggestive of:

 

trigeminal neuralgia.temporomandibular joint dysfunction syndrome.  Correcttemporal arthritis.a normal finding.

 

Explanation:

Facial asymmetry can be seen in temporomandibular joint dysfunction syndrome (TMJ).

On ophthalmoscopic examination, optic atrophy appears:

 

 

Current and Emerging Trends in Healthcare

Current and Emerging Trends in Healthcare

Administrators must continuously seek opportunities to increase the profitability of their practice or facility. Throughout this course, you will develop a plan to integrate a current and emerging trend in health care in your organization. Your Executive Summary to integrate a current or emerging solution into your organization will include the following: Current and Emerging Trends in Healthcare

ORDER A FREE-PLAGIARISM PAPER HERE

  • Week 1: Topic Research/Selection and Literature Review
  • Week 2: Organizational Assessment
  • Week 3: Project Plan
  • Week 4: Implementation Plan
  • Week 5: Evaluation Plan

Week 1: Topic Research/Selection and Literature Review

Part 1: Topic Research and Selection

Begin this process by researching what health care organizations are doing or attempting to do to increase profitability. Remember, profitability can be improved from many different angles. A nonexclusive list of potential ways would be adding additional services, decreasing costs, increasing the amount of services that are provided, or implementing a quality improvement program that qualifies for incentive monies. Check trade journals or professional discussion boards, or reach out to existing health care managers.

Part 2: Literature Review

Perform a literature review of the solution that you have decided to apply to your organization. The review will encompass several articles, and at least 1 of the articles must be peer-reviewed. Look at current material (within the last 2 years). Remember, a literature review includes a summary of the information that you found that is relevant to your topic as well as an APA reference for each resource that you reviewed. Current and Emerging Trends in Healthcare

Identifying what topic to perform research on should be accomplished quickly because you will need sufficient time to perform your literature review. Review a minimum of 10 documents. The review should be between 10–15 pages, including the bibliography. Use APA format. Current and Emerging Trends in Healthcare

Extroversion Essay Paper

Extroversion Essay Paper

Extroversion is highly connected to the need for stimulation in a way that is similar to ADHD. If a pill were developed to make people extroverted should some people start taking it, what are the risks and benefits?

Topic 4 DQ 2
As we have discovered in our readings this week, there are many risks and benefits that come along with extraversion. If a pill were developed to make people extroverted should some people start taking it, one risk would be in regard to the ability of the person to study for a test. Burger (2017) states of extroverts that, “Unless the study material is particularly exciting, the extravert will probably take a number of breaks, look around for distractors, and generally have a difficult time keeping his or her mind on the task” (p. 239). This, in turn, would likely make them get low scores on the tests and not be a very good student in general. If a significant amount of the population were to become extraverted through this pill, that could lead to more distracted students, which would not make them adequately prepared for the class material or the workforce Extroversion Essay. Like I used to say in Ethical Thinking in the Liberal Arts last semester, the world would seemingly not be a better place if this conclusion became true. Another risk related to being an extravert would be impulsivity. As Burger (2017) notes, “Extraverts are also more impulsive than introverts.  Extroversion Essay Paper. Extraverts are more likely to act on the spur of the moment, and this impulsivity can cause problems” (p. 241). I actually chose to take this class impulsively because I wanted to give myself more to do during COVID-19. That does not necessarily qualify me as an extravert, however. If a significant amount of the population took this pill and became impulsive, we would likely have a lot of people running around making decisions on the fly, which would not be an optimal circumstance for society. On the other hand, there are also benefits to extraversion. Extroverts have been found to have increased happiness, according to Burger: “Although introverts may have difficulty understanding this at first, researchers find that on average, extraverts report higher levels of happiness than introverts” (p. 240) Extroversion Essay. There are several reasons for why this is the case. Extraverts tend to socialize more than introverts, extraverts may be more sensitive to rewards than introverts, and extraverts are more likely than introverts to seek out activities they feel will be fun. Moreover, “high levels of extraversion are also linked to high levels of overall well-being” (p. 240). I interpret this as due to their social needs being met through being with many other people. These are the risks and benefits that extraverted people would possess.Burger, J.M. (2017). Personality10th ed. Mason, OH: Cengage Learning. Extroversion Essay Paper

ORDER   A PLAGIARISM FREE PAPER   NOW

Topic 4 DQ 2
I personally find it fascinating that ADHD is related to extroversion. According to our textbook the left brain and the right side of the brain work in different ways. The left identifies more as the extrovert while the right side can be in relation with the introvert trait. If we took this into account the pill would technically be rewiring the brain. Therefore, an advantage of such a pill would make it easier to study the brain in certain aspects. Biological psychologists might be able to narrow down a specific gene or pattern of the brain that indicates the likelihood of becoming depressed or an introvert. If the pill was monitored and not overdosed; another advantage would be creating more well-rounded people. Where people lack in characteristics would have the opportunity to be made up due to the pill. Extroversion Essay Although, I can see benefits there are a few disadvantages I would notice. First, I believe God made everyone unique and beautiful just the way they are. If he wanted someone with ADHD to completely change how they are he would have made them that way. God created us in imagine of himself; therefore, God in ways has ADHD. Why would we not want to be like God? Why completely get rid of the image of God within us? Another disadvantage is everyone would be the same Extroversion Essay. The world would have lack of diverse personalities and opinions. It could lead to too many cooks in the kitchen as my grandmother used to say. If we all have our own opinions and they must be heard then we will struggle to get anything done productively. Extroversion Essay Paper

ORDER   A PLAGIARISM FREE PAPER   NOW

Topic 4 DQ 2
Hi everyone,The concept of extraversion and introversion were covered in depth in this topic’s reading material. I do believe that this question depends on what an introvert wants to gain from taking a pill like this. In situations where an introvert wants to gain social experience, or gain self-confidence being in these situations then yes, a pill causing more extraverted personality traits would be helpful Extroversion Essay. When looking at extraverts, they carry themselves in a different way than introverts do. The negative side effects of taking a pill like this for an introvert, would potentially give them too much energy and social capabilities that would take away from things they previously liked doing, such as reading a book with a cup of tea, or doing yoga on the patio. Introverts are often time much more at peace with themselves, making them more comfortable spending time alone, and enjoying it. A pill causing more extraverted traits would potentially take away that peace and tranquility.

 

 

Topic 4 DQ 2

Extroversion is highly connected to the need for stimulation in a way that is similar to ADHD. If a pill were developed to make people extroverted should some people start taking it, what are the risks and benefits? Extroversion Essay

Good Evening Class,

If a pill were developed to make people extroverted I do not believe people should start taking it. It does seem plausable, as our textbook reviewed alpha waves the brain gives off when it experiences different emotions. So if researchers can study brain waves and determine that extroverts experience more of their alpha waves on the left side of the brain in correlation to positive emotions, then create a pill to help introverts be more extroverted? If our personality is infact genetically inherited and passed down to us through our parents, and their parents and so on, why would an introvert want to be more of an extrovert, and vice versa. I know that many people think of people they know and say things like why are they so friendly, or why are they so positive all the time. Or some get frustrated and want their friends or family members to be more social or outgoing. This shouldn’t cause someone to want to be something that they aren’t.

I believe that if it has taken a century and longer to study the human personality, and still there are so many luming questions, and going studies, the brain and how a persons personality develops is a very intricate part of a person. Something of this magnitude should be cared for with a certain high level of caution and care. A persons personality is a very important part of what makes them who they are. Making a pill to change them into something else just seems wrong. The ramifications of doing this to me could be very damaging and not worth the risk Extroversion Essay Paper.

Topic 4 DQ 2

If a pill were to be developed to make people extroverted, the results could vary from very minimal to extremely drastic. Extroverts are the happiest when around people and when they are able to express their feelings (“Extroversion and Introversion,” 2020). However, introverts enjoy solitude and their imagination (“Extroversion and Introversion,” 2020). If many introverts began to take a pill to help them become extroverted, it poses the threat of completely altering their mind. They are programmed a certain way and altering their personality could dramatically impact their mental state. In addition, another risk is that if there is an abundance of extroverted people who feel the need to interact with others simultaneously, there would never be a balance of energy. On the other hand, the benefit of some introverts taking a pill to become extroverted would be that there would be more social interaction in the world, which would potentially reduce loneliness and depression  Extroversion Essay Paper

Reference:

Extroversion and introversion. (2020). Columbia Electronic Encyclopedia, 6th Edition, 1.

Professional Development Objectives

Professional Development Objectives

In Week 1, you developed your practicum professional development objectives. How have your experiences thus far in the practicum contributed to your growth as nurse leader-manager or nurse informaticist? What difficulties, if any, have you experienced, and how might this affect the achievement of your objectives?

In this Discussion, you assess your progress toward fulfilling your practicum professional development objectives and consider how you could enhance or alter your activities to achieve your aims. Professional Development Objectives

ORDER A FREE-PLAGIARISM PAPER HERE


To prepare:

  • Reflect on the practicum professional development objectives you developed and outlined in your Practicum Professional Experience Plan in Week 1.
  • Keeping in mind the practicum activities you have engaged in thus far, consider the following questions:
    • How have these activities helped to promote your professional development?
    • Are you satisfied with your progress toward meeting your objectives? If not, what will you do to ensure you achieve them before the end of your Practicum Experience? As a reminder, you must complete all of your practicum hours on or before Day 5 of Week 11.
    • What challenges or unexpected opportunities have arisen at your practicum site? How has this affected your professional development?
  • Think about the experiences you may have in the forthcoming weeks. Do you foresee any particular challenges on the horizon? If so, what is your plan for addressing those challenges?
  • Think about the time you have spent with your Preceptor. How has this time enhanced or changed your understanding of the role and functions of the nurse leader-manager or nurse informaticist? Professional Development Objectives

Radical Behaviorism Assignment

Radical Behaviorism Assignment

Skinner’s “radical behaviorism” proposes that behavior, rather than cognitive processes, should be the primary topic of study in the field of psychology. As bold of a statement as this is, I can argue that it is true. Behaviorism was proposed by a trailblazing psychologist who goes by the name of B.F. Skinner. He states that the study of conscious mental life simply did not exist and that it was time for the field of psychology to join others as a natural science (Moore, 2011). By doing so, he’s transitioning from the subjective point of view to objective which requires measurement through direct observation. Radical behaviorism was founded during a period of modernism where multiple scholars and individuals transitioned away from the traditional way of thinking (Moxley, 2004) Behaviorism Essay. Cognitive processes was the first study completed in the field of psychology and it had researchers consumed for many years. However, during the period of modernism, behaviorism was introduced and suggested that cognitive processes were not a thing because they could not be directly observed.  Radical Behaviorism Assignment

Moore, J. (2011). Behaviorism. The Psychological Record, 61(3). 449-463. https://doi.org/10.1007/BF03395771 Credit Line: Behaviorism by Moore, J., in The Psychological Record, Vol. 61/ Issue 3. Copyright 2011 by Denison University. Reprinted by permission of Denison University via the Copyright Clearance Center. Licensed in 2021.

Moxley, R. A. (2004). Pragmatic selectionism: The philosophy of behavior analysis. The Behavior Analyst Today, 5(1), 108-125. https://doi.org/10.1037/h010013Discussion: Behaviorism

Photo Credit: Vitezslav Vylicil – stock.adobe.com

One skill of an applied behavior analyst is collaborating to identify negative behaviors an individual is exhibiting and then developing treatments for those behaviors (e.g., strategies, interventions, etc.) to help them change the behaviors and derive improved outcomes for themselves. Radical behaviorism posits there is a science of behavior that has shown the environment as a significant cause of behavior and that experiential factors play a primary role in determining behavior (Cooper et al., 2020). The strategies and interventions of the applied behavior analyst represent efforts to change maladaptive behavior by systematically employing techniques that control circumstances in which negative behaviors arise and helping individuals learn and apply new skills.

ORDER   A PLAGIARISM FREE PAPER   NOW

For example, if a child in a classroom is repeatedly leaving their seat during instruction (i.e., maladaptive behavior), an applied behavior analyst would work with the child to identify the factors contributing to the maladaptive behavior and then develop strategies or interventions to generate more appropriate behavior by the child. Radical Behaviorism Assignment

For this week’s Discussion, you will consider your perspectives on radical behaviorism and its learning principles.

Reference:
Cooper, J. O., Heron, T. E., & Heward, W. L. (2020). Applied behavior analysis (3rd ed.). Pearson. 

To Prepare

  • Review the Learning Resources for this week as well as the required weekly media program to gain an understanding of radical Behaviorism Essay
  • Think about whether you concur with the learning principles of radical behaviorism.

By Day 4 of Week 3

Post an explanation of whether you agree or disagree with the following statement, and why or why not:

  • Skinner’s “radical behaviorism” proposes that behavior, rather than cognitive processes, should be the primary topic of study.

Read your colleagues’ postings.

Note: For this discussion, you are required to complete your initial post before you will be able to view and respond to your colleagues’ postings. Begin by clicking on the To Participate in this Discussion link, then select Create Thread to complete your initial post. Remember, once you click on Submit, you cannot delete or edit your own posts, and cannot post anonymously Behaviorism Essay. Please check your post carefully before clicking on Submit!

By Day 6 of Week 3

Respond to at least two colleagues’ posts by expanding on each colleague’s rationale for why they agree or disagree with Skinner’s proposal. Justify your response.

Be sure to support your posts and responses with specific references to behavior-analytic theory and research. In addition to the Learning Resources, search the Walden Library and/or internet for peer-reviewed articles to support your posts and responses. Use proper APA format and citations, including those in the Learning Resources.

Return to this Discussion in a few days to read the responses to your initial posting. Note what you have learned and/or any insights that you have gained because of your colleagues’ comments  Radical Behaviorism Assignment .

I agree with Skinner’s radical behaviorism approach because I believe that behavior can describe the function of a consequence much more appropriately than the mental state of an individual. Skinner suggests that “behavior is a function of our environmental circumstances and its context” (Cooper, Heron, and Heward, 2020). In other words, whenever we act or behave, we are influenced by what is taking place within that moment and within that environment. It is when we understand the context of what is happening and the environment that it is happening in that we are able to correlate our behaviors and our responses to the consequences of what has occurred.  Moore (2011) suggests that the “basis of behavior is not simply concerned with the way something acts, but also what is going on within the environment that is manipulating the behavior”. Because our cognitive process typically expresses the cause of our behaviors, it is my opinion that the cause of a behavior is not understood without first understanding the function. I believe that when we are born, we go through different innate experiences that help us process our surroundings and inform us on how to interact with them. Moore (2011) also explains how the function of our “behavior is relative to variables at the level of phylogeny, ontogeny, or culture”. It also “reflects on the factors that are naturally within us and those that we experience within our environment” (Moore, 2011).

References

Cooper, J. O., Heron, T. E., & Heward, W. L. (2020). Applied behavior analysis (3rd ed.). Pearson.

Moore, J. (2011). Behaviorism. The Psychological Record, 61(3), 449–463. https://doi.org/10.1007/BF03395771   Behaviorism Essay

PICO ANALYSIS

PICO ANALYSIS

A PICO analysis is used to pose a focused clinical question to which you find appropriate evidence-based answers. The PICO question should include the patient or population (P), anticipated intervention (I), comparison group or current standard (C), and outcome desired (O). In this Assignment, you develop a question related to dementia, delirium, or depression. Through your PICO analysis, you explore various resources and examine current evidence to answer the question you develop. PICO ANALYSIS

ORDER A FREE-PLAGIARISM PAPER HERE

To prepare:

  • Select one of the following disorders as your topic: dementia, delirium, or depression.
  • Review the guidelines in the “Literature Review Matrix” document in this week’s Learning Resources.
  • Think about a research question around your issue as indicated in Part I: PICO Analysis of Research Topic.
  • Consider the resources you will use, search terms and criteria, and Boolean search strings as indicated in Part II: Search Strategy.
  • Using the Walden Library and other appropriate databases, locate five articles related to your PICO question. At least one article must be a systematic review. All of the articles should be primary sources.
  • Reflect on the five articles you selected as indicated in Part III: Analysis of Literature. Consider the conceptual framework/theory, main finding, research method, strengths of study, weaknesses, and level of evidence for each article.
  • Consider how to use the summaries in Part III to create an evidence table. Use this evidence table to determine appropriate treatment options for patients who present with the disorder you selected as your topic. PICO ANALYSIS

To complete:

  • Formulate a question around the disorder you selected as indicated in Part I: PICO Analysis of Research Topic.
  • Identify the resources you will use, search terms and criteria, and Boolean search strings as indicated in Part II: Search Strategy.
  • Summarize the five articles you selected as indicated in Part III: Analysis of Literature. Describe the conceptual framework/theory, main finding, research method, strengths of study, weaknesses, and level of evidence for each article.
  • Create an evidence table based on the article summaries in Part III. Describe appropriate treatment options for patients based on this evidence table. PICO ANALYSIS

 

Apea Heent Assignment

APEA HEENT

Question:

The function of the auditory ossicles is to:

 

transmit the light reflex to the light cone.transform sound vibrations into mechanical waves for the inner ear. Correctto capture sound waves from the external ear for transmission into the middle ear.to separate the inner ear from the middle ear.

 

Explanation:

The function of the auditory ossicles is to transform sound vibrations into mechanical waves for the inner ear

Question:

A 35-year-old patient complains of vertigo accompanied by nausea and vomiting. Examination reveals bilateral diplopia and an unsteady gait. These symptoms could be suggestive of:

 

an arrhythmia.a neurological condition. Correctan inner ear infection. Incorrectorthostatic hypotension. APEA HEENT.

 

Explanation:

Vertigo symptoms associated with neurologic conditions include: ataxia, diplopia, and dysarthria. Symptoms associated with cardiovascular conditions and vertigo include arrhythmias, orthostatic hypotension, vasovagal stimulation, lightheadedness, weakness, or presyncope. Apea Heent Assignment

Question:

A 60-year-old was concerned about a yellowish colored lesion above her right eyelid. Findings revealed a slightly raised yellowish, well circumscribed plaque along the nasal area of her right eyelid. This finding is most consistent with:

 

a pinguecula.a chalazion.episcleritis.xanthelasma. Correct

 

Explanation:

Slightly raised, yellowish, well-circumscribed plaques appearing along the nasal area of one or both eyelids are consistent with lipid disorders and called xanthelasma. Pinguecula refer to harmless, yellowish, triangular nodules in the bulbar conjunctiva on either side of the iris. A chalazion is a nontender nodule usually on the underside of the eyelid. Episcleritis is an ocular inflammation of the episcleral vessels. APEA HEENT.

Question:

Assessment of a patient’s visual acuity resulted in 20/200 using the Snellen eye chart. This means that:

 

at 200 feet the patient can read printed information that a person with normal vision could read at 20 feet.at 20 feet the patient can read printed information that a person with normal vision could read at 200 feet. Correctthe patient has normal visual acuity.the patient may not be able to read so he should be tested with the picture or “E” eye charts.

 

Explanation:

Visual acuity that is corrected to 20/200 constitutes legal blindness. The larger the number under 20, the worse the visual acuity. If this is a new finding, the patient needs ophthalmologic evaluation.

Question:

Findings following assessment of a person’s left eye gaze include impaired movements when attempting to look upward, downward, or inward. This condition is most consistent with:

 

a conjugate gaze.left cranial nerve III (oculomotor) paralysis Correctcranial nerve IV (trochlear) paralysis.cranial nerve VI (abducens) paralysis.

ORDER   A PLAGIARISM FREE PAPER   NOW

Explanation:

With a left cranial nerve III paralysis, upward, downward, or inward movements are impaired. In conjugate or normal gaze, the normal movement of the two eyes appears simultaneously in the same direction to bring something into view. With a left cranial nerve VI paralysis, a person’s gaze would include eyes conjugate when looking to the right, esotropia (one or both eyes turn inward) appears in the left eye when looking straight ahead, and esotropia is maximum in the left eye when looking to the left. The left eye is unable to look down when turned inward in a left cranial nerve IV paralysis.

Question:

Findings following assessment of a person’s eye gaze include both eyes moving in the same direction simultaneously. APEA HEENT. This condition is most consistent with:

 

a conjugate gaze. Correctleft cranial nerve III (oculomotor) paralysis cranial nerve IV (trochlear) paralysis.cranial nerve VI (abducens) paralysis.

 

Explanation:

In conjugate or normal gaze, the normal movement of the two eyes appears simultaneously in the same direction to bring something into view. With a left cranial nerve VI paralysis, a person’s gaze would include eyes conjugate when looking to the right, esotropia (one or both eyes turn inward) in the left eye when looking straight ahead, and esotropia is maximum in the left eye when looking to the left. With a left cranial nerve III paralysis, upward, downward, or inward movements are impaired. The left eye is unable to look down when turned inward in a left cranial nerve IV paralysis.

Question:

A patient was diagnosed as being farsighted. The term for this condition is:

 

hyperopia. Correctmyopia.strabismus.astigmatism.

 

Explanation:

Myopia, nearsightedness, occurs when light rays focus anterior to the retina. Hyperopia, farsightedness, occurs when light rays focus posterior to the retina. Strabismus, heterotropia, is a condition in which the eyes are not properly aligned with each other. In astigmatism, light rays do not focus correctly on the retina. This causes blurriness. Apea Heent Assignment

Question:

A buildup of excess fluid around the periphery of the eye orbits is known as:

 

episcleritis.pinguecula.ptosis.periorbital edema. Correct

 

Explanation:

An accumulation of fluid around the periphery of the eye orbits is known as periorbital edema.

Question:

In order to visualize the opening of Stensen’s duct, examine the:

 

dorsal surface of the tongue.area beneath the mandible at the angle of the jaw.buccal mucosa opposite the second molar. Correctsmall openings along the sublingual fold under the tongue.

 

Explanation:

The largest salivary gland is the parotid gland and it lies within the cheeks in front of the ear extending from the zygomatic arch down to the angle of the jaw. Its duct, Stensen’s duct, runs forward to an opening on the buccal mucosa opposite the second molar. If blood comes out through Stensen’s duct when it is palpated, this could suggest parotid cancer. If pus is expelled, it suggests suppurative parotitis. With mumps, the orifice of the Stensen duct appears erythematous and enlarged. The submandibular gland is the size of a walnut. It lies beneath the mandible at the angle of the jaw. Wharton’s duct runs up and forward to the floor of the mouth and opens at either side of the frenulum. The smallest, the almond-shaped sublingual gland, lies within the floor of the mouth under the tongue. It has many small openings along the sublingual fold under the tongue. APEA HEENT.

Question:

What connects the middle ear to the nasopharynx?

 

The tympanic membraneThe proximal end of the eustachian tube CorrectThe malleusThe ossicles

 

Explanation:

The proximal end of the eustachian tube connects the middle ear to the nasopharynx.

Question:

The fleshly projection of the earlobe is known as the:

 

lobule. Correcttragus.auricle.helix.

 

Explanation:

The fleshy projection of the earlobe is known as the lobule. The auditory canal opens behind a nodular protuberance that points backward over the entrance to the canal. This is known as the tragus. The auricle is made of cartilage covered by skin and has a firm elastic consistency. The auricle has a prominent curved outer ridge known as the helix.

Question:

When examining the pupils, the left pupil is noted to be fixed and dilated to light and near accommodation. This condition may be suggestive of:

 

a tonic pupil. Incorrectoculomotor nerve (CN III) paralysis. CorrectHorner’s syndrome.Argyll Robertson pupils.

 

Explanation:

Paralysis of the oculomotor cranial nerve (CN III), the dilated pupil is fixed to light and near accommodation. Ptosis and lateral deviation of the eye are usually present. When the pupil is large, regular, and usually unilateral and the reaction to light is severely reduced and slowed, or even absent, this condition is referred to as a tonic pupil or Adele’s pupil. In Horner’s syndrome, the affected pupil reacts briskly to light and near effort but the pupil is small. The pupils in Argyll Robertson condition appear small and irregular shaped and accommodate but do not react to light.

Question:

A deposit of uric acid crystals appearing as hard nodules on the helix or antihelix is termed:

 

a keloid.a tophi. Correcta cutaneous cyst.chondrodermatitis.

 

Explanation:

The antihelix divides the helix from the lobe. Tophi is a deposit of uric acid crystals (that appear commonly in patients with chronically elevated uric acid levels) on the helix or antihelix. They can also appear near the joints, hands, or feet. It is also seen in chronic tophaceous gout. A firm, nodular, hypertrophic mass of scar tissue extending beyond the area of injury is classified as a keloid. A cutaneous cyst has a characteristic dome shaped lump in the dermis forming a benign closed firm sac attached to the epidermis. This lesion was formerly known as a sebaceous cyst. A chronic inflammatory lesion that starts as a painful, tender papule on the helix or antihelix is known as chondrodermatitis.

Question:

On the outer ear, anterior and parallel to the helix, is a curved prominence known as the:

 

Antihelix CorrectHelixAuricleTragus

 

Explanation:

The antihelix is a curved prominence that is parallel and anterior to the helix and is part of the auricle. The external ear consists of the auricle and ear canal. The auricle is made of cartilage covered by skin and has a firm elastic consistency. The auricle has a prominent curved outer ridge known as the helix. The ear canal opens behind a nodular protuberance that points backward over the entrance to the canal. This is called the tragus. APEA HEENT.

Question:

On examination of the pupils, both are round but the right pupil appears larger than the left and reacts much slower to light. This condition may be indicative of:

 

a tonic pupil. Correctoculomotor nerve (CN III) paralysis. IncorrectHorner’s syndrome.Argyll Robertson pupils.

 

Explanation:

When the pupil is large, regular, and usually unilateral and the reaction to light is severely reduced and slowed, or even absent, this condition is referred to as a tonic pupil or Adele’s pupil. Paralysis of the oculomotor cranial nerve (CN III), the dilated pupil is fixed to light and near accommodation. Ptosis and lateral deviation of the eye are usually present. In Horner’s syndrome, the affected pupil reacts briskly to light and near effort but the pupil is small. The pupils in Argyll Robertson condition appear small and irregular shaped and accommodate but do not react to light.

Question:

Leukoplakia was noted during an exam of the mouth. This symptom may be:

 

a normal finding.precancerous. Correctassociated with periodontal disease.consistent with gingivitis.

 

Explanation:

Leukoplakia are thickened white patches located on any area of the mouth. These patches cannot be rubbed off. Most are not serious but some can be considered precancerous. Therefore, they should be evaluated. These lesions are not considered normal findings. Periodontal disease usually includes an infection of the gums and may involve the teeth. Generally, the infection causes redness and swelling but not white patches. APEA HEENT.

Question:

The majority of people who present with non-24 hour sleep-wake disorder are:

 

legally blind.attention deficit.color blind.totally blind. Correct

 

Explanation:

The majority of people who present with non-24 hour sleep-wake disorder are totally blind. This chronic circadian rhythm sleep disorder is defined as a condition of insomnia related to abnormal synchronization between the 24-hour light -dark cycle and the endogenous circadian rhythms of sleep and wake cycles.

Question:

The most common cause of bacterial pharyngeal infections in children is:

 

Corynebacterium.Chlamydia.mononucleosis.group A beta-hemolytic Streptococcus. Correct

 

Explanation:

Pharyngitis is caused by swelling between the tonsils and the larynx. Most sore throats are caused by colds, the flu, Coxsackie virus or mononucleosis. Bacteria that cause pharyngitis include group A beta-hemolytic Streptococcus, and less commonly, Corynebacterium, gonorrhea, and Chlamydia can cause sore throat. APEA HEENT.

Question:

When inspecting the neck for the thyroid gland, slightly tilt the patient’s head back, and using tangential lighting directed downward from the tip of the patient’s chin, inspect the:

 

region above the thyroid cartilage.region below the cricoid cartilage. Correctarea along the sternomastoid border.area along the anterior edge of the trapezius.

 

Explanation:

When inspecting the neck for the thyroid gland, slightly tilt the patient’s head back and using tangential lighting (light coming in from the side at a right angle) directed downward from the tip of the patient’s chin, inspect the region below the cricoid cartilage; located between the thyroid cartilage and the thyroid gland. The area of sternomastoid border allows palpation of the superficial cervical nodes. The posterior cervical lymph nodes are located at the anterior edge of the trapezius. APEA HEENT.

Question:

A condition in which the eyes are not properly aligned with each other is termed:

 

hyperopia.myopia.strabismus. Correctastigmatism.

 

Explanation:

Strabismus, heterotropia, is a condition in which the eyes are misaligned. “Crossed-eyed”, “wall eye”, or “lazy eye” are all associated with strabismus. Hyperopia, farsightedness, occurs when light rays focus posterior to the retina. Myopia, nearsightedness, occurs when light rays focus anterior to the retina. In astigmatism, light rays do not focus correctly on the retina. This causes blurriness.

Question:

Round or oval shaped lesions surrounded by erythematous mucosa and noted on an area of the oral mucosa may be:

 

leukoplakia. aphthous ulcers. CorrectKoplik’s spots.ulcerative gingivitis.

 

Explanation:

Aphthous ulcers can appear anywhere on the buccal mucosa or tongue. They usually appear as round or oval ulcers, can be white or yellowish gray in color, and are surrounded by a halo of reddened mucosa. They are usually painful. Leukoplakia presents as thickened white patches anywhere on the oral mucosa. Koplik’s spots appear in the early stages of measles (rubeola). They appear as small white specks that resemble grains of salt on a red background on the buccal mucosa. They are not usually painful. Ulcerative gingivitis is a painful form of gingivitis that is characterized by the development of ulcers in the interdental papillae. If untreated they can become necrotizing along the gum margins and appear as erythematous ulcers.

Question:

On physical exam, an abnormal Rinne test might indicate:

 

impaired physical mobility.impaired visual acuity.impaired hearing ability. Correctimpaired swallowing ability.

 

Explanation:

The Rinne test uses a tuning fork to compare air conduction to bone conduction and so is used to assess hearing. The other choices are not assessed using the Rinne test.

Question:

Ophthalmoscopic examination reveals dark specks noted between the fundus and the lens. These specks are most likely:

 

superficial retinal hemorrhages. Incorrectcataracts.drusen.vitreous floaters. Correct

 

Explanation:

Vitreous floaters may be seen as dark specks or strands between the fundus and the lens. They could be a symptom of retinal detachment, retinal tear, or bleeding within the eye. Superficial retinal hemorrhages appear as small, linear, flame-shaped, red streaks in the fundus and are seen in hypertension, papilledema, and occlusion of the retinal vein. Cataracts are opacities in the lens of the eye. Drusen appear as yellowish round spots posteriorly between the optic disc and the macula and are associated with cellular debris.

Question:

A 60- year-old patient presents with severe, deep left eye pain. Findings reveal dilated and fixed left pupil and the cornea is cloudy. There is no ocular discharge noted. These findings are most likely consistent with:

 

acute iritis.corneal injury. Incorrectcorneal infection.acute angle closure glaucoma. APEA HEENT. Correct

 

Explanation:

With acute angle closure glaucoma, the pain is described as severe, aching, and deep. The pupils are dilated and fixed and the cornea appears steamy or cloudy. If an increase in intraocular pressure is present, this would be an emergency situation. Acute iritis presents with a moderate aching pain deep within the eye. The pupils are small and irregular and vision is decreased and photophobia is usually present. The cornea is clear or slightly cloudy with injection confined to the corneal limbus. This is considered an emergency and is usually related to Herpes zoster infection or tuberculosis. Corneal injury or infection usually presents with watery or purulent ocular discharge and the pain is severe. Apea Heent Assignment

Question:

The function of the labyrinth in the inner ear is to:

 

assist with air conduction.maintain equilibrium. Correctmaintain acoustic transmission.capture sound waves.

ORDER   A PLAGIARISM FREE PAPER   NOW

Explanation:

The inner ear has 2 main functions: hearing and balance. The cochlear system is dedicated to hearing and the vestibular system is dedicated to balance. The labyrinth is part of the semicircular canals and the vestibular system and is responsible for balance. APEA HEENT.

Question:

The curved outer ridge of the auricle of the ear is known as the:

 

AntihelixHelix CorrectAuricle IncorrectTragus

 

Explanation:

The auricle has a prominent curved outer ridge known as the helix. The external ear consists of the auricle and ear canal. The auricle is made of cartilage covered by skin and has a firm elastic consistency. The antihelix is a curved prominence that is parallel and anterior to the helix. The auditory canal opens behind a nodular protuberance that points backward over the entrance to the canal, known as the tragus.

Question:

A person who has been blind since birth presents for a physical exam. Expected findings of the pupillary reaction when light is shown would be:

 

constriction of both pupils.dilation of both pupils.no reaction from either pupil.it depends. Correct

 

Explanation:

It depends on the type of blindness. Pupillary reaction is controlled by the muscles which are innervated by nerves. As long as there is no damage to the nerve or the muscle, the pupil will still contract or dilate.

Question:

A six-year-old complains that something is in her left eye. There is a red raised area of the left lid. There is redness and tenderness of the eye and tearing. These findings are consistent with:

 

blepharitis.conjunctivitis. Incorrecta corneal ulcer.a hordeolum. Correct

 

Explanation:

A hordeolum, or stye, is an infection of the sebaceous gland of the eyelid. Symptoms include a raised area of the lid, pain, redness, tenderness, possible photophobia, tearing, and a sensation of a foreign body in the eye. Conjunctivitis presents with a reddened conjunctiva and presence of a watery or purulent discharge depending on the etiology. A corneal ulcer produces a visible area of ulceration. Blepharitis presents with an erythematous eyelid margin with a mucous discharge, crusting, and scaling of the eyelid margins.

Question:

Ophthalmoscopic examination of the retina reveals AV tapering. This appears as if the:

 

vein “winds” down on either side of the artery. Correctvein is twisted on the distal side of the artery.vein crosses beneath the artery.vein stops abruptly on either side of the artery.

 

Explanation:

When the arterial walls lose their transparency, changes appear in the arteriovenous crossings. Decreased transparency of the retina probably contributes to AV nicking and AV tapering. In tapering, the vein appears to taper or “wind” down either side of the artery. In AV nicking, the vein appears to stop abruptly on either side of the artery. In the normal eye, the vein appears to cross beneath the artery. With banking, the vein appears to be twisted on the distal side of the artery and forms a dark wide knuckle appearance.

Question:

A patient presents with complaints of burning, itching, tearing, and some pain in the eye. Findings reveal red, scaly, greasy flakes and thickened, crusted lid margins. APEA HEENT. This would be suggestive of:

 

a chalazion. blepharitis. Correcta hordeolum.dacryocystitis.

 

Explanation:

Red, scaly, greasy flakes and thickened, crusted lid margins are consistent with blepharitis. Symptoms include burning, itching, tearing, foreign body sensation, and some pain. Chalazion is an inflammatory lesion that develops in a meibomian tear gland. Hordeolum is a localized staphylococcal infection of the hair follicles at the lid margin. Dacryocystitis is infection and blockage of lacrimal sac and duct.

Question:

Ophthalmoscopic examination of the fundus reveals small, rounded, slightly irregular red spots embedded in the retina. These findings are consistent with:

 

superficial retinal hemorrhages.preretinal hemorrhages.microaneurysms. Incorrectdeep retinal hemorrhages. Correct

 

Explanation:

Deep retinal hemorrhages appear as small, rounded, slightly irregular red spots and are sometimes called dot or blot hemorrhages. They occur in a deeper layer of the retina than flame-shaped hemorrhages. Diabetes is a common cause. Superficial retinal hemorrhages appear as small, linear, flame-shaped, red streaks in the fundi and are seen in hypertension, papilledema, and occlusion of the retinal vein. Preretinal hemorrhages lie anteriorly between the retina and the vitreous and are typically larger than retinal hemorrhages. APEA HEENT. These hemorrhages obscure any underlying retinal vessel. Microaneurysms present as tiny, round, red spots commonly seen in and around the macular area. These are classic in diabetic retinopathy.

Question:

Redness, bleeding, pain, and swelling of the gums is most likely:

 

stomatitis.gingivitis. Correctleukoplakia.aphthous ulcers.

 

Explanation:

Swelling, pain, erythema, and bleeding of the gums are symptoms of gingivitis. Stomatitis refers to inflammation of the mouth. Leukoplakia presents as thickened white patches anywhere in the oral mucosa. Aphthous ulcers can appear anywhere on the buccal mucosa or tongue. They usually appear as round or oval ulcers, can be white or yellowish gray in color, and surrounded by a halo of reddened mucosa APEA HEENT.

Question:

Ophthalmoscopic examination of the retina reveals a normal arteriovenous crossing. This appears as if the:

 

vein tapers down on either side of the artery.vein is twisted on the distal side of the artery.vein crosses beneath the artery. Correctvein stops abruptly on either side of the artery.

 

Explanation:

In the normal eye, the vein appears to cross beneath the artery. When the arterial walls lose their transparency, changes appear in the arteriovenous crossings. Decreased transparency of the retina probably contributes to AV nicking and AV tapering. In AV nicking, the vein appears to stop abruptly on either side of the artery. In tapering, the vein appears to taper down either side of the artery. With banking, the vein appears to be twisted on the distal side of the artery and forms a dark wide knuckle appearance.

Question:

Otosclerosis is an example of a(n):

 

conductive hearing loss. Correctsensorineural hearing loss.mixed hearing loss.acquired hearing loss.

 

Explanation:

Otosclerosis is a hereditary disorder of the labyrinthine capsule in which abnormal bone growth occurs around the ossicles resulting in fixation of the stapes. The stapes does not vibrate which reduces sound transmission to the inner ear. This produces a conductive hearing loss.

Question:

Which of the following findings in a preschooler would indicate the need for further evaluation?

 

Intelligible speech by 24 months of ageVariation in quality of speech pattern and toneResponds to facial expressions and gestures rather than to verbal explanations CorrectLooks at people when they speak

 

Explanation:

A child who responds to facial expressions and gestures rather than to verbal explanations is probably expressions rather than verbal clues. These children may have a hearing deficit that needs further evaluation. The other choices are normal behaviors for the preschooler.

Question:

A 50- year-old patient complains of being unable to read the hymnal at church. This describes:

 

hyperopia. Incorrectmyopia.presbyopia. Correctastigmatism.

 

Explanation:

Presbyopia may begin in the 5th or 6th decade. Symptoms include diminished ability to focus on near objects. Hyperopia, farsightedness, occurs when light rays focus posterior to the retina. Myopia, nearsightedness, occurs when light rays focus anterior to the retina. Strabismus, heterotropia, is a condition in which the eyes are misaligned. “Crossed-eyed”, “wall eye”, or “lazy eye” are all associated with strabismus. APEA HEENT.

Question:

The gradual loss of vision with a change in color and size of the optic disc is referred to as:

 

macular degeneration. Incorrectglaucoma. Correctcataracts.retinoblastoma

 

Explanation:

With glaucoma, there is a change in the color and size of the optic disc resulting in a gradual loss of vision. With macular degeneration, there is a loss of vision in the central visual fields because of damage to the retina. Peripheral vision remains intact. Cataracts usually appear as a cloudiness or opacity in the lens. Retinoblastoma is a rapidly developing cancer of the retina and an absent red reflex is noted on ophthalmoscopic exam.

Question:

In order to examine the tongue, ask the patient to stick out his tongue and with the examiner’s right hand:

 

stimulate the patient to cough.pull the tongue downward and push down with the finger on the left hand to elicit the gag reflex.grasp the tip of the tongue, gently pull it to the left side , and inspect the side of the tongue. Correctinspect it for symmetry.

 

Explanation:

In order to examine the tongue, ask the patient stick out his the tongue and with the examiner’s right hand, grasp the tip of the tongue with a gauze and gently pull it to the patient’s left. Inspect the side of the tongue, then palpate it with the left hand, feeling for any induration. Reverse the procedure for the other side. Inspecting for tongue symmetry checks function of cranial nerve XII, hypoglossal. Stimulating the patient to cough and eliciting the gag reflex do not have anything to do with the tongue examination.

Question:

A 30-year-old patient presents with complaints of seeing double in the right eye. Examination reveals diplopia in the right eye when the left eye is closed. This may be suggestive of a:

 

problem in the cornea. Correctproblem in the optic disc.palsy of cranial nerve III or IV.palsy of cranial nerve III or VI. Incorrect APEA HEENT.

 

Explanation:

Problems in the cornea or lens present as unilateral diplopia in one eye when the other eye is closed. Diplopia could be suggestive of weakness or paralysis of the extraocular muscles or of a tumor in the cerebellum or brainstem. Vertical diplopia results from damage to cranial nerves (CN) III or IV. If the diplopia is horizontal, it could be suggestive of paralysis of one or more extraocular muscles resulting form damage to cranial nerves (CN) III or VI. Problems in the cornea or lens present as unilateral diplopia. The optic disc is not associated with diplopia.

Question:

One cause of nasal septum perforation may be:

 

nasal polyps.intranasal use of cocaine. Correctcystic fibrosis.chronic sinusitis.

 

Explanation:

Perforation of the nasal septum could be caused by trauma, surgery, and intranasal use of cocaine or amphetamines. Nasal polyps obstruct air flow but there is no relationship to nasal septum perforation. Cystic fibrosis or chronic sinusitis are not associated with nasal perforation.

Question:

A 30-year-old patient presents with a moderate “aching” in his right eye. Findings reveal a small and irregular shaped right pupil. The cornea appears cloudy with a slight erythematous area around the corneal limbus. There is no ocular discharge noted. APEA HEENT. These findings are consistent with:

 

acute iritis. Correctcorneal injury. Incorrectcorneal infection.acute angle closure glaucoma.

 

Explanation:

Acute iritis presents with a moderate aching deep within the eye. The pupils are small and irregular and vision is decreased and photophobia is present. The cornea is clear or slightly cloudy with injection confined to the corneal limbus. This is considered an emergency and is usually related to Herpes zoster infection or tuberculosis. Corneal injury or infection usually presents with watery or purulent ocular discharge and severe eye pain. With acute angle closure glaucoma, the pain is severe, aching, and deep, but the pupils are dilated and fixed and the cornea appears steamy or cloudy. If an increase in intraocular pressure is present in conjunction with these findings, this would be an emergency situation.

Question:

Sudden bilateral and painless visual loss is rare but can be associated with all the following except:

 

cholinergics.anticholinergics.steroids.chemical exposure. Correct

 

Explanation:

Certain medications are associated with sudden bilateral, painless visual loss. These medication classes include cholinergic, anticholinergics, and steroids. Exposure to chemicals or radiation could result in bilateral painful visual loss. APEA HEENT.

Question:

The Weber test uses a tuning fork to test hearing. The frequency range closest to that of conversational speech would be one with a frequency of:

 

256 Hz.512 Hz. Correct800 Hz.1000Hz.

 

Explanation:

The Weber hearing test screens for unilateral conductive hearing loss with a tuning fork. The tuning fork is measured in frequencies of 256 Hz or 512 Hz. These frequencies fall within the range of conversational speech. The ideal frequency for the Weber test is 256 Hz. A frequency of 512 Hz is the ideal frequency for the Rinne hearing test.

Question:

On ophthalmoscopic examination, optic atrophy appears:

 

pink and hyperemic.yellowish orange to creamy pink.pale.white. Correct

 

Explanation:

In optic atrophy, there is death of the optic nerve fibers. This leads to loss of the tiny disc vessels giving a white appearance. A pink and hyperemic disc is seen in patients with papilledema. The normal disc appears yellowish-orange to creamy pink and the disc vessels are tiny with disc margins that appear sharp. A pale disc with a backward depression is characteristic of glaucomatous cupping.

Question:

Ophthalmoscopic examination of the retina reveals AV banking. This appears as if the:

 

vein tapes down on either side of the artery.vein is twisted on the distal side of the artery. Correctvein crosses beneath the artery. Incorrectvein stops abruptly on either side of the artery.

 

Explanation:

When the arterial walls lose their transparency, changes appear in the arteriovenous crossings. Decreased transparency of the retina probably contributes to AV nicking and AV tapering. In AV nicking, the vein appears to stop abruptly on either side of the artery. In tapering, the vein appears to taper down either side of the artery. In the normal eye, the vein appears to cross beneath the artery. With banking, the vein appears to be twisted on the distal side of the artery and forms a dark wide knuckle appearance. APEA HEENT.

ORDER   A PLAGIARISM FREE PAPER   NOW

Question:

Causes of sensorineural hearing loss include all of the following except:

 

aging.loud noises over prolonged periods of time. Incorrectperforated tympanic membrane. Correctacoustic neuroma.

 

Explanation:

A perforated tympanic membrane is an example of a cause of a conductive hearing loss. Sensorineural hearing loss involves disorders of the cochlear nerve, cochlea, and an interruption in the neuronal impulse transmission to the brain. Causes include: exposure to loud noises, inner ear infections, acoustic neuroma, aging, and congenital and/or familial disorders.

Question:

What visual acuity constitutes legal blindness?

 

Visual acuity of 20/80 or worse bilaterally.Visual acuity of 20/200 or worse in the better eye with corrected lens. CorrectVisual acuity of 20/200 in the better eye without corrected lens.Visual acuity of 20/100 with corrected lens.

 

Explanation:

A person is usually considered legally blind when visual acuity in the better eye, corrected by lens, is 20/200 or worse. One is also considered legally blind if visual acuity is 20 degrees or less in the better eye and the vision is constricted. They are said to have “tunnel” vision in this case. Apea Heent Assignment

Question:

An example of a cause of conductive hearing loss in children would be:

 

prolonged use of tobramycin. Incorrectthe presence of a peanut in the ear for three weeks. Correctcongenital rubella syndrome.maternal history of Herpes infection.

 

Explanation:

Conductive hearing loss is the most common type in children. It occurs when the transmission of sound through the external or middle ear is blocked. It may be temporary or permanent, unilateral or bilateral. It may also be caused by physical abnormalities present from birth, but more commonly, it begins during childhood as the result of middle ear infections, perforation of the eardrum, impacted earwax or objects present in the ear canal. The other choices are examples of sensorineural hearing losses. APEA HEENT.

Question:

Miosis is a term used to describe:

 

the shape of the pupils.constriction of the pupils. Correctdilation of the pupils.symmetry of the pupils.

 

Explanation:

Miosis refers to constriction of the pupils, mydriasis refers to dilation. Miosis is not associated with the shape or symmetry of the pupils.

Question:

If a patient complains of seeing flashing lights across the field of vision, this could be:

 

a normal response if around bright lights.a retinal detachment. Incorrectdetachment of the vitreous from the retina. Correctlesion in the visual pathways.

 

Explanation:

Flashing lights or new vitreous floaters suggest detachment of vitreous from the retina. A detached retina would present with sudden loss of vision. With a lesion in the visual pathway, one may visualize fixed defects such as a scotoma. At any rate, this is not a normal finding even if the person was around bright lights. APEA HEENT.

CARE MANAGEMENT

CARE MANAGEMENT

In the United States, managed care is becoming an increasingly popular method of administering healthcare. It influences the clinical behavior of providers, as it combines the payment and delivery of healthcare into a single system, the purpose of which is to control the cost, quality, and access of healthcare services for a single bracket of health plan enrollees (Scutchfield, Lee, & Patton, 1997). CARE MANAGEMENT

ORDER A FREE-PLAGIARISM PAPER HERE

Yet, managed care often evokes strong or negative reactions from healthcare providers because they are paid a fixed amount for treating their patients, regardless of the actual cost, which may influence their level of efficiency. This can challenge the relationships between doctors and patients (Claxton, Rae, Panchal, Damico, & Lundy, 2012; Sekhri, 2000).

Research managed care’s inception and study some examples. Be sure to investigate the perspectives about managed care from the vantage of both healthcare providers and patients. You can use the following keywords for your research—United States managed care, history of managed care, and managed care timeline.

Based on your research, answer the following questions in a 8- to 10-page Microsoft Word document:

  • What are the positive and negative aspects of managed care? Analyze the benefits and the risks for both providers and patients, and how providers should choose among managed care contracts. Conclude with your analysis and recommendations for managed care health plans. Your response should include answers to the following questions:
    • Summarize the history of when, how, and why managed care was developed.
    • Define and discuss each type of managed care organization (MCO)—health maintenance organization (HMO), preferred provider organization (PPO), and point of sale (POS).
    • Explain the positive and negative aspects, respectively, of managed care organization from the provider’s point of view—a physician and a healthcare facility—and from a patient’s point of view.
    • Explain the three types of incentives for providers for efficiency in the delivery of healthcare services. Explain who bears the financial risk—the provider, the patient, or the managed care organization.
    • Offer your recommendations, to accept or decline, for patients considering managed care health plans, with your rationale for each. CARE MANAGEMENT

References:

Claxton, G., Rae, M., Panchal, N., Damico, A., & Lundy, J. (2012). Employer Health
           Benefits Annual 2012 Survey
. Retrieved from http://ehbs.kff.org/pdf/2012/
           8345.pdf

Sekhri, N. K. (2000). Managed care: The US experience. Retrieved from http://www.
           who.int/bulletin/archives/78%286%29830.pdf

Scutchfield F. D., Lee, J., & Patton, D. (1997). Managed care in the United States.
           Journal of Public Health Medicine, 19(3), 251–254. Retrieved from http://
           jpubhealth.oxfordjournals.org/content/19/3/251.full.pdf

Support your responses with examples.

Cite any sources in APA format. CARE MANAGEMENT

Dermatology quiz Assignment paper

A microscopic examination of the sample taken from a skin lesion indicates hyphae. What type of infection might this indicate?

A child with a sandpaper-textured rash probably has:

A 40-year-old female patient presents to the clinic with multiple, painful reddened nodules on the anterior surface of both legs. She is …. These are probably … with her history of:

A patient is … with tinea pedis. A microscopic examination of the sample taken from the infected area would likely demonstrate:

When can a child with chickenpox return to daycare?

A patient with a primary case of scabies was probably …:

The nurse practitioner examines a patient who has had poison ivy for 3 days. She asks if she can spread it to her family members. The nurse practitioner replies:

Which chronic skin disorder primarily affects hairy areas of the body? Dermatology quiz Assignment paper

A patient with diabetes has right anterior shin edema, erythema, warmth, and tenderness to touch. This developed over the past 3 days. There is no visible pus. What is the most likely … to consider?

The agent commonly used to treat patients with scabies is permethrin. How often should it … applied to eradicate scabies? Dermatology quiz.

Impetigo is characterized by:

A 60-year-old patient is noted to have rounding of the distal phalanx of the fingers. What might have caused this?

A patient has suspected scarlet fever. He likely has a sandpaper rash and:

A patient with a positive history of a tick bite about 2 weeks ago and erythema migrans has a positive ELISA for Borrelia burgdorferi. The Western blot is positive. How should he … managed?

A patient will … taking oral terbinafine for fingernail fungus. The NP knows that:

A skin lesion that is a solid mass is … as a:

The nurse practitioner is examining a 3-month-old infant who has normal development. She has identified an alopecic area at the occiput. What should … done? Dermatology quiz.

The most common form of skin cancer is:

The nurse practitioner identifies satellite lesions in a 6-month-old infant. These are:

A patient who is at high risk for skin cancer should:

An example of a premalignant lesion that develops on sun-damaged skin is:

Which of the following areas of the body has the greatest percutaneous absorption?

The main difference between cellulitis and erysipelas is the:

Most cases of atopic dermatitis exacerbation are treated with

A 6-year-old patient with sore throat has coryza, hoarseness, and diarrhea. What is the likely etiology?

A pregnant mother in her first trimester has a 5-year-old who has Fifth Disease. What implication does this have for the mother? Dermatology quiz Assignment paper

A 74-year-old woman is … with shingles. The NP is deciding how to best manage her care. What should … prescribed?

An adolescent has acne. The nurse practitioner prescribed a benzoyl peroxide product for him. What important teaching point should … given to this adolescent regarding the benzoyl peroxide?

A 70-year-old is … with multiple cherry angiomas. The nurse practitioner knows that:

A patient has seborrheic dermatitis. Which vehicle would … most appropriate to use in the hairline area to treat this?

A patient with eczema asks for a recommendation for a skin preparation to help with xerosis. What should the NP respond?

A topical treatment for basal cell carcinoma is:

The most common place for basal cell carcinoma to … found is the:

An infant is … with diaper dermatitis. Satellite lesions are visible. This should … treated with a:

A 3-year-old female had a fever of 102° F for the last 3 days. Today she woke up from a nap and is afebrile. She has a maculopapular rash. Which statement is true? Dermatology quiz.

A 71-year-old female presents with a vesicular rash that burns and itches. Shingles is …. An oral antiviral:

A skin disorder has a hallmark finding of silvery scales. What word below describes this common condition?

A child has 8-10 medium brown café au lait spots > 1 cm in diameter. The differential diagnosis should include:

A low-potency topical hydrocortisone cream would … most appropriate in a patient who has been … with:

A patient is found to have koilonychia. What laboratory test would … prudent to perform?

A 23-year-old male appears in clinic with the following lesion on his trunk. This lesion is usually associated with:

A 28-year-old has thick, demarcated plaques on her elbows. Which features are suggestive of psoriasis?

ORDER   A PLAGIARISM FREE PAPER   NOW

Which of the following skin lesions in an older adult is a premalignant condition?

A patient presents with plaques on the extensor surface of the elbows, knees, and back. The plaques are erythematous and thick, silvery scales are present. This is likely:

A 15-year-old male has worked this summer as a lifeguard at a local swimming pool. He complains of itching in the groin area. He is … with tinea cruris. The nurse practitioner is likely to identify:

A 68-year-old female adult with pendulous breasts complains of “burning” under her right breast. The nurse practitioner observes a malodorous discharge with mild maceration under both breasts. What is this?

An example of a first-generation cephalosporin used to treat a skin infection is:

A patient has a “herald patch” and is … with pityriasis rosea. Where is the “herald patch” most commonly found? Dermatology quiz. Dermatology quiz Assignment paper

A 16-year-old has been … with Lyme disease. Which drug should … used to treat him?

A 9-year-old female has presented to your clinic because of a rash on the left, upper area of her anterior trunk. She is embarrassed and very reticent to lift her blouse because her nipple will … exposed. How should the NP proceed?

Hand-foot-and-mouth disease and herpangina:

A 74-year-old male patient has sustained a laceration to his foot. His last tetanus shot was more than 10 years ago. He has completed the primary series. What should … recommended?

An adolescent takes isotretinoin for nodulocystic acne. She is on oral contraceptives. Both were prescribed by the dermatologist. The adolescent presents to your clinic with a sinus infection. Her temperature is 99.5° F and her blood pressure is 160/100 mmHg. How should this … managed?

Johnson is a 74-year-old who presents with a pearly-domed, nodular-looking lesion on the back of the neck. It does not hurt or itch. What is a likely etiology?

A 16-year-old male has nodulocystic acne. What might have the greatest positive impact in managing his acne?

Which of the following antibiotics may increase the likelihood of photosensitivity?

Which of the following lesions never blanches when pressure is applied?

An older adult patient has been … with shingles on the right lateral aspect of her trunk. It initially appeared yesterday. It is very painful. How should she … managed? Dermatology quiz.

A young child has developed a circumferential lesion on her inner forearm. It is slightly raised, red and is pruritic. It is about 2.5 cm in diameter. This is probably related to:

The primary therapeutic intervention for patients who present with hives is:

The lesions seen in a patient with folliculitis might … filled with:

Which vehicle is least appropriate in a patient who has atopic dermatitis?

A patient has been … with scabies. What is the medication of choice to treat this?

The American Cancer Society uses an ABCDE mnemonic to help patients develop awareness of suspicious skin lesions. What does the “B” represent?

A “herald patch” is a hallmark finding in which condition?

A patient has used a high-potency topical steroid cream for years to treat psoriasis exacerbations when they occur. She presents today and states that this cream “just doesn’t work anymore.” What word describes this?

A patient has been … with MRSA. She is allergic to sulfa. Which medication could … used to treat her?

Topical 5-fluorouracil (5-FU) is used to treat:

A patient was burned with hot water. He has several 2-3 cm fluid-filled lesions. What are these termed?

A child received a burn on his chest from a cup of hot coffee. On examination, the injured area appeared moist, red to ivory white in color, and features blisters. It is painful to touch. This burn would … classified as a:

What finding is most characteristic of shingles? A patient reports that he found a tick on himself about 2 weeks ago. He presents today with a red circle and a white center near where he remembers the tick bite. He did not seek treatment at that time. Today he complains of myalgias and arthralgias. Which laboratory test can … used to help diagnose Lyme disease?

The best way to evaluate jaundice associated with liver disease is to observe:

The term caput succedaneum refers to:

A 4-year-old has been … with measles. The nurse practitioner identifies Koplik’s spots. These are:

A patient calls your office. He states that he just came in from the woods and discovered a tick on his upper arm. He states that he has removed the tick and the area is slightly red. What should he … advised?

A wound has the following characteristics; partial thickness loss of dermis, a shallow open ulcer with red/pink bed, and no evidence of sloughing. What stage of pressure ulcer does this describe?

ORDER   A PLAGIARISM FREE PAPER   NOW

A patient has been in the sun for the past few weeks and has developed darkened skin and numerous 3-6 mm light-colored, flat lesions on his trunk. What is the likely etiology?

What advice should … given to a parent who has a child with Fifth Disease?

A 6-year-old has been … with Lyme disease. Which drug should … used to treat him?

A patient presents with small vesicles on the lateral edges of his fingers and intense itching. On close inspection, there are small vesicles on the palmar surface of the hand. What is this called?

A patient has 10 cm of well demarcated erythema on his lower leg that is raised and warm to touch. He had an abrupt onset of lower leg pain, and fever that began 36 hours ago. What is this? Dermatology quiz Assignment paper

What is the proper technique to safely remove a tick from a human? Dermatology quiz.

A skin lesion fluoresces under a Wood’s lamp. What microscopic finding is consistent with this?

A 23-year-old male appears in clinic with the following nonpruritic lesion on his trunk. He first noticed this about 3 days ago. The lesion is probably:

A patient presents to the minor care area of the emergency department after … bitten by a dog. The patient states that the dog had a tag around his neck and had been seen roaming around the neighborhood for days before the patient was bitten. The dog did not exhibit any odd behavior. How should this … managed? Patients with atopic dermatitis are likely to exhibit: Which test is NOT suitable to diagnose shingles if the clinical presentation is questionable?

A patient exhibits petechiae on both lower legs but has no other complaints. How should the NP proceed?

A 9-year-old has …. with chickenpox. A drug that should … avoided in him is: Dermatology quiz.

HEALTH CARE ETHICS

HEALTH CARE ETHICS

Discussion 1

Create an enforceable contract in any department of a health care organization. How can a contract be legally valid? When is a contract voidable?  What happens when one of the parties cannot perform the obligations written in the contract?

ORDER A FREE-PLAGIARISM PAPER HERE

Discussion 2


Develop a plan for compliance with the Sarbanes-Oxley Act (SOX) in a hospital setting. How can employees become knowledgeable with the major provisions of SOX? How can top executives comply with the major provisions? How can conflicts of interests in the organization be observed?  HEALTH CARE ETHICS


Using the South University Online Library or the Internet, research the two major study designs–cohort and case-control–used in health care research. Find a research article on any topic in health care.

Based on your research, please express your views on the following:

  • Which study design has been used for research?
  • What is your learning from the chosen article and its study design?
  • Is the design selected for the research, best for this particular study? Why or why not?

In addition to the above, your response should also include answers to the following questions:

  • Although cohort studies are very powerful, case-control studies tend to be more popular. Do you agree with the statement? Why or why not?
  • How does the cohort study design differ from the case-control study design?
  • What is essentially the main use or purpose of the cohort study design and case-control study design?
  • When it is best to use the cohort study design and when is it best to use the case-control study design?
  • What characteristics of cohort study design make it important in health care research?
  • Cohort studies can be retrospective or prospective. What makes a cohort study retrospective or prospective?
  • please cite sources HEALTH CARE ETHICS